Converting triple integral coordinates

Click For Summary
The discussion focuses on setting up a triple integral for the function (x^2 + y^2) dV within a solid sphere of radius R. The user initially establishes the integral in rectangular coordinates but is advised to ensure the correct order of integration as dz dy dx. The conversation then shifts to cylindrical coordinates, where the user correctly identifies the bounds and integrand as r^3 dz dr dtheta. Finally, the user seeks clarification on setting up the integral in spherical coordinates, confirming the appropriate bounds and integrand as (p^4)(sin(φ))^3 dp dφ dtheta. The thread emphasizes the importance of proper coordinate transformations in triple integrals.
619313
Messages
8
Reaction score
0
1. consider the triple integral (x^2 +Y^2) dV where it is bounded by a solid sphere of radius R. Set up the integral using rectangular coordinatesI tried setting this up with the bounds [ -sqrt(R^2-x^2-Y^2) <= Z <= sqrt(R^2-x^2-Y^2) ,
-R <= X <= R , -sqrt(R^2-x^2) <= Y <= sqrt(R^2-x^2) ] am I on the right path?
 
Last edited:
Physics news on Phys.org
Yes, but make sure you have them in the correct order when you write down the integrals.
 
Fantastic! Yea I know that it goes dz dy dx
now How about if I rewrite to cylindrical coordinates?
0<= theta <= 2pi , 0<=r<=R , -sqrt(R^2-r^2)<=z<= sqrt(R^2-r^2) with the integrand being r^3 dz dr dtheta
 
Yes, you've got it.
 
first, thanks so much for the help

just to make sure I fully understand all of these triple integrals, to set it up in spherical I should get
0<= theta<= 2pi, 0<= phi <= pi, 0<= p <= R,
where the integrand is (p^4) (sin(FI))^3 dp dphI dtheta
 
Yes. Very nice. And welcome to the Physics forum.
 
Question: A clock's minute hand has length 4 and its hour hand has length 3. What is the distance between the tips at the moment when it is increasing most rapidly?(Putnam Exam Question) Answer: Making assumption that both the hands moves at constant angular velocities, the answer is ## \sqrt{7} .## But don't you think this assumption is somewhat doubtful and wrong?

Similar threads

  • · Replies 2 ·
Replies
2
Views
2K
  • · Replies 9 ·
Replies
9
Views
2K
  • · Replies 1 ·
Replies
1
Views
2K
Replies
6
Views
2K
  • · Replies 7 ·
Replies
7
Views
2K
  • · Replies 3 ·
Replies
3
Views
867
Replies
4
Views
2K
  • · Replies 21 ·
Replies
21
Views
3K
Replies
3
Views
2K
  • · Replies 3 ·
Replies
3
Views
1K